0 Daumen
438 Aufrufe


ich wollte nun mit der Definition der konvergenten Folge zeigen, dass $$a_n := \sqrt{n+1} - \sqrt{n-1}$$ gegen 0 konvergiert. Also

$$| a_n - 0 | = | a_n | < \epsilon $$

$$| \sqrt{n+1} - \sqrt{n-1} | = | \frac{  (\sqrt{n+1}+\sqrt{n-1})(\sqrt{n+1}-\sqrt{n-1})  }{ \sqrt{n+1} + \sqrt{n-1} } | = | \frac{2}{\sqrt{n+1} + \sqrt{n-1}} | = \frac{2}{\sqrt{n+1}+\sqrt{n-1}} < \epsilon$$

$$\Leftrightarrow \frac{2}{\sqrt{n+1} + \sqrt{n+1}} = \frac{2}{2 \sqrt{n+1}} = \frac{1}{\sqrt{n+1}}< \frac{2}{\sqrt{n+1}+\sqrt{n-1}} < \epsilon$$

$$\Leftrightarrow \sqrt{n+1} > \frac{1}{\epsilon} \Leftrightarrow n+1 > \frac{1}{\epsilon^2} \Leftrightarrow n > \frac{1}{\epsilon^2} - 1$$

Also muss $$n_0 > \frac{1}{\epsilon^2} - 1$$ sein.

Das stimmt aber für Epsilon = 1 nicht...

$$\epsilon = 1 \Rightarrow \frac{1}{1^2} - 1 = 0 \Rightarrow n_0 = 1 \Rightarrow | a_1 | = \sqrt{2} = 1,41... > 1 = \epsilon$$

Wo liegt der Fehler? Ich konnte bei obigen Überlegungen auch nach längerem Hinschauen nichts feststellen...

Danke,

Thilo
Avatar von 4,3 k

Du schätzt in der ersten Äquivalenz in die falsche Richtung ab.

Die Bestimmte das n0 für die Minorante 1/(n+1)0.5, eine Majorante wär angebrachter.

Mal ganz abgesehen davon, dass hier mit Äquivalenzen zu arbeiten nicht wirklich sinnvoll ist.

Du vergisst auch die Quantoren vor der Aussage mitzunehmen.

1 Antwort

0 Daumen
Hallo Thilo87,

um für

\( \frac{2}{\sqrt{n+1}+\sqrt{n-1}} < \epsilon \)

eine Vereinfachung zu finden, kannst du den linken Term nach oben abschätzen durch

\( \frac{2}{\sqrt{n+1}+\sqrt{n-1}} < \frac{2}{\sqrt{n-1}+\sqrt{n-1}} = \frac{1}{\sqrt{n-1}} \).

Findest du nun ein \( \epsilon \), sodass

\( \frac{2}{\sqrt{n+1}+\sqrt{n-1}} < \frac{2}{\sqrt{n-1}+\sqrt{n-1}} < \epsilon \),

so kannst du gemäß deiner Gleichung umstellen:

\( \frac{1}{\sqrt{n-1}} < \epsilon \),

\( n > \frac{1}{\epsilon^2} + 1 \)

und erhältst ein plausibles Ergebnis.

MfG

Mister

PS: Wie im Kommentar von Anonym angekündigt, haben wir hier also statt der Minorante eine Majorante für den Ausdruck \( \frac{2}{\sqrt{n+1}+\sqrt{n-1}} \) benutzt. \( \frac{2}{\sqrt{n-1}+\sqrt{n-1}} = \frac{1}{\sqrt{n-1}} \) ist diese Majorante.
Avatar von 8,9 k

Ein anderes Problem?

Stell deine Frage

Willkommen bei der Mathelounge! Stell deine Frage einfach und kostenlos

x
Made by a lovely community